• Anúncio Global
    Respostas
    Exibições
    Última mensagem

Dúvida

Dúvida

Mensagempor diegodalcol » Sex Out 02, 2009 18:14

Boa tarde pessoal, estou com uma duvida em uma resolução de eum exercicio de antenas e propagação e gostaria de ver se alguém pode me ajudar, não sei esse aqui é o lugar certo senão for por favor me desculpem.
Segue abaixo a resolução.

[tex]Zen=50*[\frac{50+j75+j50*tg 12,56*0,05}{50+j(50+j75)*tg 12,56*0,05}](em radianos)
Zen=50*[\frac{50+j75+j36,3}{50+j*(36,3+j54,45)}Zen=50*[\frac{50+j111,3}{50-54,45+j36,3}]Zen=50*[\frac{122,01{e}^{j*1,15}}{36,57{e}^{j*1,69}}]Zen=50-3,34{e}^{-j0,54}Zen=167{e}^{-j0,54}Zen=143,23-j85,86


grato.
diegodalcol
Novo Usuário
Novo Usuário
 
Mensagens: 7
Registrado em: Qui Mai 22, 2008 13:06
Área/Curso: Estudante
Andamento: cursando

Voltar para Números Complexos

 



  • Tópicos relacionados
    Respostas
    Exibições
    Última mensagem

Quem está online

Usuários navegando neste fórum: Nenhum usuário registrado e 3 visitantes

 



Assunto: Unesp - 95 Números Complexos
Autor: Alucard014 - Dom Ago 01, 2010 18:22

(UNESP - 95) Seja L o Afixo de um Número complexo a=\sqrt{8}+ i em um sistema de coordenadas cartesianas xOy. Determine o número complexo b , de módulo igual a 1 , cujo afixo M pertence ao quarto quadrante e é tal que o ângulo LÔM é reto.


Assunto: Unesp - 95 Números Complexos
Autor: MarceloFantini - Qui Ago 05, 2010 17:27

Seja \alpha o ângulo entre o eixo horizontal e o afixo a. O triângulo é retângulo com catetos 1 e \sqrt{8}, tal que tg \alpha = \frac{1}{sqrt{8}}. Seja \theta o ângulo complementar. Então tg \theta = \sqrt{8}. Como \alpha + \theta = \frac{\pi}{2}, o ângulo que o afixo b formará com a horizontal será \theta, mas negativo pois tem de ser no quarto quadrante. Se b = x+yi, então \frac{y}{x} = \sqrt {8} \Rightarrow y = x\sqrt{8}. Como módulo é um: |b| = \sqrt { x^2 + y^2 } = 1 \Rightarrow x^2 + y^2 = 1 \Rightarrow x^2 + 8x^2 = 1 \Rightarrow x = \frac{1}{3} \Rightarrow y = \frac{\sqrt{8}}{3}.

Logo, o afixo é b = \frac{1 + i\sqrt{8}}{3}.